Back to Articles

DrBeen Question (4001)

A 42 years old patient presents to your clinic for a complete physical because he recently moved here from another state. He feels in good health. He has never smoked, doesn’t have diabetes, and is sure that he does not have hypertension either. He exercises regularly and eats a healthy diet.
On physical examination, you notice a slow rising pulse. His apical beat is displaced to left 6th intercostal space in the mid-axillary line. A harsh mid-systolic murmur is heard best on the right 2nd intercostal space. You notice that the murmur is radiating towards the carotids. Murmur intensity decreases with Valsalva maneuver and increases with squatting. What is the most probable diagnosis in this patient?

A. Aortic stenosis.
B. Pulmonary valve stenosis.
C. Ventricular septal defect.
D. Patent foramen ovale.
E. Hypertrophic obstructive cardiomyopathy (HOCM).

 

Reference ID: 4001

Check the answer here: https://www.drbeen.com/blog/answer-drbeens-question-4001/

Loading...